4
$\begingroup$

Notation: Here $\mathcal Y_t$ denotes the natural filtration of the process $Y_t$, and $\{\cdot\}$ denotes the fractional part of a real number.

This question concerns detecting the presence (or otherwise) of a brief, periodic impulse signal in the presence of noise.

Let $X$ be a uniform random variable on $[0, 1]$, and $0 < \delta < 1$ a real number. Consider the solution $Y$ to the SDE

$$dY_t = A \, \mathbf 1_{[0, \delta]} (\{t - X\}) \ dt + \sigma \,dW_t,$$

with $W$ a standard one dimensional Brownian motion independent of $X$, and $A, \sigma > 0$ constants. We assume the starting condition $Y_0 = 0$ a.s.

In this setup, $\delta$ is the duration of the impulse signal, $\sigma$ is the strength of the noise, and $A$ is the amplitude of the signal.

Question:

I believe that given enough time, we can ascertain with almost certainty whether or not the signal is present. More precisely, we should have $\mathbb E[X| \mathcal Y_t] \to X$ almost surely and in $L^1$ as $t \to \infty$, but this seems quite difficult to prove.

In any case, if this is true, I would like to quantify the effects of the parameters on the result. Can we estimate $\mathbb E[|\mathbb E[X | \mathcal Y_t] - X|]$ in terms of the parameters $\delta, A, \sigma$?

References: The ideas in the paper Seperating signal from noise (Liv, Peled, Peres) may be helpful.

$\endgroup$
6
  • $\begingroup$ As soon as t > 2 $Y_t = A \delta + \sigma W_t$ so I don’t think you get any information about X after that time. $\endgroup$ Commented Aug 19, 2024 at 22:01
  • $\begingroup$ @mike I don’t think that holds, perhaps something in the problem set up is not clear? $\endgroup$ Commented Aug 20, 2024 at 1:00
  • $\begingroup$ I believe explicitly we have $$Y_t = A\delta ([t] - 1)_+ + \max(\{t\}, X) + \max(\{t\}, X + \delta - 1) - X + \sigma W_t.$$ Which is messier than I expected! Maybe there is a nicer way to write it. $\endgroup$ Commented Aug 20, 2024 at 1:18
  • $\begingroup$ Although I don’t think the explicit formula is of much help anyway to answer the problem. $\endgroup$ Commented Aug 20, 2024 at 1:19
  • $\begingroup$ … sorry typo, it is $$Y_t = A[\delta ([t] - 1)_+ +\max(\{t\}, X) + \max(\{t\}, X + \delta - 1) - X] + \sigma W_t.$$ $\endgroup$ Commented Aug 20, 2024 at 1:25

1 Answer 1

2
+500
$\begingroup$

Does the following make any sense?

For $n = 1, 2, \ldots$ and $t \in [0, 1)$ let $$ Z_n(t) = \frac{1}{n} \sum_{k = 0}^{n - 1} (Y_{k + t} - Y_k) . $$ Then $Z_n$ is measurable with respect to $\mathcal Y_{n + t}$ and $$ Z_n(t) = \frac{1}{n} \sum_{k = 0}^{n - 1} \biggl(\sigma W_{k + t} - \sigma W_k + A \int_0^t \mathbb 1_{[0,\delta]}(\{n + s - X\}) ds\biggr) . $$ Since the integral does not depend on $n$, in fact we have $$ Z_n(t) = \frac{\sigma}{n} \sum_{k = 0}^{n - 1} (W_{k + t} - W_k) + A \int_0^t \mathbb 1_{[0,\delta]}(\{s - X\}) ds . $$ The first term almost surely converges to zero (in fact uniformly with respect to $t \in [0, 1)$). Thus, $$ Z(t) = \lim_{n \to \infty} Z_n(t) = \int_0^t \mathbb 1_{[0,\delta]}(\{s - X\}) ds $$ is measurable with respect to $\mathcal Y_\infty$.

Knowing $Z(t)$, one easily reconstructs $X$ (e.g. the unique $t$ such that $Z'(t^-) = 0$ and $Z'(t^+) = 1$ is equal to $X$), so it turns out that $X$ is measurable with respect to $\mathcal Y_\infty$. In particular, $\mathbb E[X | \mathcal Y_t]$ is a martingale which converges to $\mathbb E[X | \mathcal Y_\infty] = X$ almost surely and in $L^1$.

$\endgroup$
5
  • 1
    $\begingroup$ Hm, I will check this in detail soon, interesting idea. $\endgroup$ Commented Oct 14, 2024 at 21:56
  • $\begingroup$ I assume you meant $W_{k+t} - W_k$ everywhere, so I am modifying it, but please let me know if I am mistaken! $\endgroup$ Commented Oct 14, 2024 at 23:03
  • $\begingroup$ The solution checks out IMO, very nicely done. $\endgroup$ Commented Oct 14, 2024 at 23:05
  • $\begingroup$ Wait a minute, there is still another question open, the one about the rate of convergence. Time permitting, I will get back to it later this week. Oh, and thanks for the edit, that is what I meant. $\endgroup$ Commented Oct 15, 2024 at 8:00
  • $\begingroup$ Yep, you could still answer that if you want. I will unaccept the answer so the problem will display as still open, but enjoy the +500 imaginary points as my show of gratitude! $\endgroup$ Commented Oct 15, 2024 at 8:55

You must log in to answer this question.

Start asking to get answers

Find the answer to your question by asking.

Ask question

Explore related questions

See similar questions with these tags.